Son İletiler

Sayfa: [1] 2 3 ... 10
1
Fantezi Geometri / IA yarıçaplı çember
« Son İleti Gönderen: geo Bugün, 12:47:55 öö »
$ABC$ üçgeninde iç açıortayların kesişim noktası $I$ olup $I$ merkezli $A$ dan geçen çember $BC$ doğrusunu $D$ ve $E$ de kesiyor. $BC+DE = AB+AC$ olduğunu gösteriniz.
2
Analiz-Cebir / Ynt: En küçük değer
« Son İleti Gönderen: Hüseyin Yiğit EMEKÇİ Dün, 06:19:40 ös »
Evet hocam haklısınız. Benim yazdığım çözümde de Bergström Eşitsizliği Cauchy Eşitsizliği'ne, Cauchy Eşitsizliği de skaler çarpım eşitsizliğine dayanmaktadır.
3
Analiz-Cebir / Ynt: En küçük değer
« Son İleti Gönderen: alpercay Dün, 05:09:53 ös »
$<,>$ iç çarpımı göstermek üzere , Titu lemma ya da Bergström eşitsizliğini kullanmadan $$<X,Y>\le |X|\cdot |Y|$$ skaler çarpım eşitsizliği ile de çözülebiliyor. Burada vektörleri $X=(\sqrt{2/a},\sqrt{3/b},\sqrt{4/c})$   ve  $Y=(\sqrt{5a},\sqrt{6b},\sqrt{7c})$ almak gerekiyor.
4
Analiz-Cebir / Ynt: En küçük değer
« Son İleti Gönderen: Hüseyin Yiğit EMEKÇİ Dün, 04:29:57 ös »
$$LHS=\dfrac{2}{a}+\dfrac{3}{b}+\dfrac{4}{c}=\dfrac{10}{5a}+\dfrac{18}{6b}+\dfrac{28}{7c}\overbrace{\geq}^{Bergström} \dfrac{\left(\sqrt{10}+\sqrt{18}+\sqrt{28}\right)^2}{5a+6b+7c}=\left(\sqrt{10}+\sqrt{18}+\sqrt{28}\right)^2$$
olarak minimum değer tayin edilebilir.
5
Analiz-Cebir / En küçük değer
« Son İleti Gönderen: alpercay Dün, 03:20:44 ös »
$a,b,c$ pozitif reel sayılar , $$5.a+6.b+7.c=1$$ olmak üzere  $$\dfrac{2}{a}+\dfrac{3}{b}+\dfrac{4}{c}$$ ifadesinin en küçük değeri kaçtır?
6
2021 / Ynt: Tübitak Lise 1. Aşama 2021 Soru 08
« Son İleti Gönderen: Hüseyin Yiğit EMEKÇİ Nisan 22, 2024, 07:41:11 ös »
Cevap: $\boxed{D}$
Yukarıdaki ispat sınav sırasında öğrenciler tarafından test taktiğiyle her kırmızı ve beyaz boyalı sayıların kendi içinde eş olmasıyla $2x^2=x$ ve $x$ sıfır olmadığından $\dfrac{1}{2}$ olarak bulunabilir. Lakin daha akla yatar bir çözüm verelim.
$a_1=x, a_2=xy, a_3=y$ olsun. Buna göre
$$a_4=y\left(1-x\right)$$
$$a_5=1-x$$
$$a_6=\left(1-x\right)\left(1-y\right)$$
$$a_7=1-y$$
$$a_8=x\left(1-y\right)$$
olarak elde edilir ve bundan sonra $a_9=x$ olduğundan dizi periyodik bir hal alır. Yani $8$'lik bir tekrar bulduk. 0 zaman
$$Toplam=25\left(\sum_{i=1}^{8}{a_i}\right)=25\left(x+xy+y+y-xy+1-x+1-x-y+xy+1-y+x-xy\right)=25.3=75$$
olarak bulunur.
7
Analiz-Cebir / Ynt: n değişkenli bir eşitsizlik {çözüldü}
« Son İleti Gönderen: Hüseyin Yiğit EMEKÇİ Nisan 21, 2024, 09:17:22 ös »
Bergström Eşitsizliği'ni kullanacak olursak

$$LHS=\sum_{cyc- j}{\dfrac{a_j^2}{2a_j-a_j^2}}\geq \dfrac{\left(\sum\limits_{cyc}{a_1}\right)^2}{2\sum\limits_{cyc}{a_1}-\sum\limits_{cyc}{a_1^2}}=\dfrac{1}{2-\sum\limits_{cyc}{a_1^2}}$$

$\sum\limits_{cyc}{a_1^2}\geq \dfrac{\left(\sum\limits_{cyc}{a_1}\right)^2}{n}=\dfrac{1}{n}$ olduğunu kullanırsak
$$LHS\geq \dfrac{1}{2-\sum\limits_{cyc}{a_1^2}}\geq \dfrac{1}{2-\dfrac{1}{n}}=\dfrac{n}{2n-1}$$
8
2024 / Ynt: Avrupa Kızlar Matematik Olimpiyatı 2024 Soru 2
« Son İleti Gönderen: geo Nisan 20, 2024, 07:45:13 ös »
$K$ nin içteğet çembere göre kuvvetinden $$KX \cdot KY = KD^2 \tag{1}$$
$K$ nin çevrel çembere göre kuvvetinden $$KB\cdot KC = KT^2 \tag{2}$$
Teğet kiriş açıdan $\angle DYX = \angle XDB$.
$\angle XBK = \angle XDB+ \angle BXD = \angle DYX + \angle DYC =\angle CYX$ olduğu için $CYXB$ kirişler dörtgenidir. $K$ nin bu kirişler dörtgeninin çevrel çemberine göre kuvvetinden $$KX \cdot KY = KB\cdot KC \tag{3}$$
$(1),(2)$ ve $(3)$ ü birleştirirsek $KT = KD$ elde ederiz.

$TD$ ile $\Omega$, $M$ de kesişsin.
$\overset{\Huge\frown}{BM} + \overset{\Huge\frown}{TC} =2\angle TDC = 2\angle DTK =\overset{\Huge\frown}{TM} =\overset{\Huge\frown}{CM} +\overset{\Huge\frown}{TC} \Longrightarrow \overset{\Huge\frown}{BM} = \overset{\Huge\frown}{CM}$
Bu durumda $AM$ doğrusu $\angle CAB$ nin açıortayıdır. Yani $A, I, M$ doğrusaldır. O halde $AI$ ile $TD$ doğruları, $\overset{\Huge\frown}{BC}$ yayının orta noktasında kesişir.
9
Sayılar Teorisi / Ynt: Güvenli Asal Sayılar
« Son İleti Gönderen: geo Nisan 20, 2024, 05:02:52 ös »
Cevap: $\boxed {(c) \ 3}$

$r>2$ ise $r=2k+1$ olacağından $p=2q+r-1=2q+2k=2(q+k)$ olacağı için çözüm gelmez.

$r=2$ için $p=2q+1$ olacaktır.
İlk birkaç $q$ değeri için çözümleri yazalım.
$q=2, p=5$,
$q=3, p=7$,
$q=5, p=11$,
$q=11, p=23$

$q>3$ olsun ve eşitliği $\bmod 6$ da inceleyelim.
$q=6k, 6k+2, 6k+4$ çift sayı oldukları için sağlamaz. $q=6k+3$ sayısı $3$ ile bölündüğü için sağlamaz.
$q=6k+1$ i yerine yazarsak $p=2q+1=12k+3=3(4k+1)$  $3$ ile bölündüğü için sağlamaz
O halde geriye sadece $q=6k+5$ şeklinde asal sayılar kalır. Yerine yazarsak $p=2q+1=12k+11$ elde ederiz.
O halde $p \equiv 5, 7, 11 \pmod {12}$ olabilir.

Not:
$p,q$; $p=2q+1$ şeklindeki asal sayılar olmak üzere $q$ sayısına Sophie Germain asalı, $p$ sayısına da güvenli asal denir.
bkz. Sophie Germain Asalları
10
2020 / Genç Balkan Matematik Olimpiyatı 2020 Soru 4
« Son İleti Gönderen: matematikolimpiyati Nisan 18, 2024, 01:56:53 öö »
$1 + \dfrac{p^q - q^p}{p + q}$ ifadesini asal sayı yapan tüm $p$ ve $q$ asal sayılarını bulunuz.

(Arnavutluk)
Sayfa: [1] 2 3 ... 10

SimplePortal 2.3.3 © 2008-2010, SimplePortal